Please confirm topic selection

Are you sure you want to trigger topic in your Anconeus AI algorithm?

Please confirm action

You are done for today with this topic.

Would you like to start learning session with this topic items scheduled for future?

Review Question - QID 108757

In scope icon M 7 D
QID 108757 (Type "108757" in App Search)
An 81-year-old woman presents to her primary care physician with a 6 month history of occasional weakness in her right arm and leg. She reports 3 such episodes that each lasted approximately 1 hour and did not leave any long-term symptoms. The patient denies numbness, tingling, pain, or changes in her speech. She has a medical history of hypertension and coronary artery disease with stable angina. Her medications include aspirin, lisinopril, amlodipine, and atorvastatin dose as moderate-intensity. She reports a 40-pack-year smoking history and occasional alcohol intake. Her temperature is 98.5°F (36.9°C), blood pressure is 142/87 mmHg, pulse is 70/min, and respirations are 14/min. She has a systolic ejection murmur best heard at the right upper sternal border and there is a carotid bruit on the left side. Her lungs are clear. Neurologic exam reveals intact cranial nerve function, 1+ deep tendon reflexes in bilateral patellae and biceps, as well as 5/5 strength and intact pinprick sensation in all extremities. Carotid ultrasound is performed and identifies 52% stenosis on the right side and 88% on the left. Which of the following is the most appropriate next step in management?